• Anúncio Global
    Respostas
    Exibições
    Última mensagem

FUVEST 2017 Segunda fase matemática

Provas
Regras do fórum

  1. Não envie somente enunciados de problemas, informe suas tentativas e dificuldades!

    Queremos que a "ajuda" represente um trabalho interativo, pois saber especificar a dúvida exige estudo.

    Serão desconsiderados tópicos apenas com enunciados, sem interação. Nosso objetivo não é resolver listas de exercícios;



  2. Para não haver má interpretação em suas postagens, especialmente na precedência das operações, utilize LaTeX, podendo ser a partir do botão "editor de fórmulas".


    Bons estudos!

FUVEST 2017 Segunda fase matemática

Mensagempor rcompany » Qui Mar 28, 2019 15:15

FUVEST 2017 matemática segunda fase com propostas de resolução.

Comentem, corrijam!



M01

Em uma competição de vólei, estão inscritos 5 times.Pelo regulamento, todos os times devem se enfrentar apenas uma vez e, ao final da competição, eles serão classificados pelo número de vitórias. Dois ou mais times com o mesmo número de vitórias terão a mesma classificação. Em cada jogo, os times têm probabilidade \frac{1}{2} de vencer.

a) Explique por que 2 times não podem empatar na classificação com 4 vitórias cada um

\text{Suponhamos que os times A e B terminem com 4 vit\'orias cada.  } \; \text{A teria vencido B e B teria vencido A, o qual \'e imposs\'ivel.}

b) Qual é a probabilidade que o primeiro classificado termine a competição com 4 vitórias?

\text{Temos }{5\choose 2}=10
\text{com cada jogo tendo 2 resultados possive\'is:}
\text{h\'a }2^{10}\text{ combina\c c\~oes poss\'iveis dos resultados dos 10 jogos}

\text{Para qualquer time }T_i\text{, um resultado do torneio que lhe d\'a 4 vit\'orias}

\text{- uma combina\c c\~ao dos resultados dos seus 4 jogos que lhe d\^e 4 vitorias: tem uma s\'o ou }{4\choose 4}
\text{- qualquer outra combina\c c\~ao de resultados dos outros 6 jogos do torneio, cujo n\'umero \'e }2^6

P\big(T_i=4\big)=\dfrac{1\times 2^6}{2^{10}}=\dfrac{1}{16}

\begin{array}{rll}P\Big(\bigcup_{i=1}^{5}(T_i=4)\Big)=&\displaystyle \sum_{i=1}^{5}P\big(T_i=4\big)&\text{j\'a que os }T_i=4\text{ s\~ao incompat\'iveis entre se}\\
=&5\times \dfrac{1}{16}&\\
\end{array}


c) Qual é a probabilidade de que os 5 times terminem empatados na classificação?

\text{Seja }T_i\text{ o n\'umero de vit\'orias do time i depois de 4 jogos.}
\text{Queremos calcular }P(\bigcap_{i=1}^{5}(T_i=2)

Primeira opção: passar pelo evento complementar. Reduziremos o número de cálculos já que aparecerão eventos incompatíveis (as combinações de T_i=3 e T_i=4)

P(\bigcap_{i=1}^{5}(T_i=2)=1-P\Bigg(\overline{\bigcap_{i=1}^{5}(T_i=2)}\Bigg)=1-P(\bigcup_{i=1}^{5}(T_i=3\cup T_i=4))
=1-P((\bigcup_{i=1}^{5}T_i=3)\cup (\bigcup_{i=1}^{5}T_i=4))\\=1-P(\bigcup_{i=1}^{5}T_i=4)-P(\bigcup_{i=1}^{5}T_i=3)+P((\bigcup_{i=1}^{5}T_i=3)\cap(\bigcup_{j=1}^{5}T_j=4)\\\\\\
P\big((\bigcup_{i=1}^{5}T_i=4)\big)=\dfrac{320}{1024}=\dfrac{5}{16}
\\\\\\
P\big((\bigcup_{i=1}^{5}T_i=3)\big)=\sum_{i=1}^{5}P(T_i=3)\\- \sum_{i\neq j}P(T_i=3\cap T_j=3)\\+\sum_{i\neq j\neq k}P(T_i=3\cap T_j=3\cap T_k=3)\\-\sum_{i\neq j\neq k\neq l}P(T_i=3\cap T_j=3\cap T_k=3\cap T_l=3)\\+ P(\bigcap_{i=1}^{5}P(T_i=3)\\
=&5\times \dfrac{{4 \choose 3}\cdot 2^6}{2^{10}}-{5 \choose 2}\times \dfrac{({3 \choose 2}{3 \choose 3}+{3 \choose 3}{3 \choose 2})\cdot 2^3}{2^{10}}\\&+{5 \choose 3}\times\dfrac{(0+{3 \choose 3}{3 \choose 3}+{3 \choose 3}{3 \choose 3}+0)\cdot 2^1}{2^{10}}+0+0\\& \text{ j\'a que 4 ou mais times n\~ao podem ter 3 vit\'orias ao mesmo tempo}\\(\text{teriamos 12 ou mais vit\'orias para 10 jogos: imposs\'ivel)}\\
=&\dfrac{1280-480+40}{1024}=\dfrac{840}{1024}=\dfrac{105}{128}\end{array}

P\Bigg(\Big(\bigcup_{i=1}^{5}T_i=3\Big)\cap\Big(\bigcup_{j=1}^{5}T_j=4\Big)\Bigg)=P\Bigg(\bigcup_{i=1}^{5}\Big(\bigcup_{j=1}^{5} (T_i=3 \cap T_j=4)\Big)\Bigg)\\
=\sum_{i=1}^{5}P\Big( \bigcup_{j=1}^{5} (T_i=3 \cap T_j=4) \Big) \text{ j\'a que os }(T_i=3\;\cap \;T_j=4)\text{ s\~ao incompat\'iveis entre eles}\\
=\sum_{i=1}^{5}\Bigg(\sum_{j=1}^{5}P\Big( T_i=3 \cap T_j=4 \Big)\Bigg)\text{ j\'a que os }(T_i=3\;\cap \;T_j=4)\text{ s\~ao incompat\'iveis entre eles}\\
=\sum_{i=1}^{5}\Bigg(\sum_{\begin{tiny}\begin{array}{c}j=1\\j \neq i\end{array}\end{tiny}}^{5}P\Big( T_i=3 \cap T_j=4 \Big)+\sum_{j=1}^{5}P\Big( T_i=3 \cap T_i=4 \Big)\Bigg)\\
=5\times \Big( 4\times \dfrac{{4 \choose 4}{3 \choose 3}\cdot 2^3}{2^{10}}+5\times 0\Big)=\dfrac{160}{1024}=\dfrac{5}{32}


\text{E ent\~ao }P\Big(\bigcap_{i=1}^{5}T_i=2\Big) = \dfrac{1024-320-840+160}{1024}= \dfrac{24}{1024}=\dfrac{3}{128}


Segunda Opção: usar os resultados possíveis dos jogos de 4 times entre eles

\text{Seja }C_4\text{ a classifica\c c\~ao por n\'umero de vit\'orias depois dos 6 jogos de 4 times entre se,}
\text{ou seja um time ainda n\~ao jogou contra ningu\'em}




\text{E ent\~ao }

\begin{array}{rl}P\Big(\bigcap_{i=1}^{5}T_i=2\Big)=&P\Big(\big(\bigcap_{i=1}^{5}T_i=2\big)\slash C_4=(2,2,1,1)\Big) \times P\big(C_4=(2,2,1,1)\big)\\
=&\dfrac{1}{16}\cdot\dfrac{384}{2^{10}}=\dfrac{1}{2^4}\cdot\dfrac{3\cdot2^7}{2^{10}}=\dfrac{3}{128}=\dfrac{24}{1024}\end{array}



Terceira opção: calcular diretamente P\Big(\bigcap_{i=1}^5T_i=2\Big)

\begin{array}{rl}P\Big(\bigcap_{i=1}^5T_i=2\Big)=&P(T_1=2)\cdot P\big(T_2=2\slash T_1=2\big)\\&\cdot P\big(T_3=2\slash (T_1=2\cap T_2=2)\big)\\&\cdot P\Big(T_4=2\slash\bigcap_{i=1}^{3}T_i=2\Big)\cdot P\Big(T_5=2\slash\bigcap_{i=1}^{4}T_i=2\Big)\end{array}

P(T_1=2)=\dfrac{{4 \choose 2}\cdot 2^6}{2^{10}}=\dfrac{384}{1024}=\dfrac{3}{8}

P(T_2=2\slash T_1=2)=\dfrac{\big(\binom{1}{3}\binom{2}{3}+\binom{2}{3}\binom{1}{3}\big)2^3}{384}=\dfrac{144}{384}=\dfrac{3}{8}


P\big(T_3=2\slash (T_1=2\cap T_2=2)\big)=\\\dfrac{\Big(2\times \big(\binom{0}{2}\binom{1}{2}\binom{2}{2}\big)+2\times\big(\binom{0}{2}\binom{2}{2}\binom{1}{2}+\binom{1}{2}\binom{1}{2}\binom{1}{2}\big)+2\times\big(\binom{1}{2}\binom{2}{2}\binom{0}{2}\big)\Big)2^{1}}{144}\tag{dasdasdasd}\\=\dfrac{(2\times 2 +2\times 10+2\times 2)2^{1}}{144}\\=\dfrac{56}{144}=\dfrac{7}{18}


P\Big(T_4=2\slash\bigcap_{i=1}^{3}T_i=2\Big)=\dfrac{24}{56}=\dfrac{3}{7}

\text{E ent\~ao }P\Big(\bigcap_{i=1}^5T_i=2\Big)=\dfrac{3}{8}\cdot\dfrac{3}{8}\cdot\dfrac{7}{18}\cdot\dfrac{3}{7}=\dfrac{3}{128}=\frac{24}{1024}



M02

Considere as funções f:\;[-\frac{\pi}{2};\frac{\pi}{2}]\rightarrow[-1;1] e g:\;[0;\pi]\rightarrow[-1;1] definidas por f(x)=\sin{x} e g(x)=\cos{x}. Sendo f e g bijetoras, existem funções f^{-1} e g^{-1} tais que f\circ f^{-1}=f^{-1}\circ f=id e g\circ g^{-1}=g^{-1}\circ g=id, em que id é a função identidade.

a) Para 0\leq \alpha\leq 1, mostre que g \circ f^{-1}(\alpha)=\sqrt{1-\alpha^2

g\circ f^{-1}(\alpha) = cos(f^{-1}(\alpha))=\sqrt{1-sin^2(f^{-1}(\alpha))} \text{ j\'a que }0\leq\alpha\leq 1\Rightarrow 0\leq f^{-1}(\alpha)\leq \frac{\pi}{2}\Rightarrow \cos{(f^{-1}(\alpha))\geq 0
\text{Ou seja } g \circ f^{-1}(\alpha)=\sqrt{1-(f\circ f^{-1}(\alpha))^2}=\sqrt{1-\alpha^2

\text{Da mesma forma mostra-se que } f \circ g^{-1}(\alpha)=\sqrt{1-\alpha^2


b) Mostre que f^{-1}(\frac{1}{2}) + g^{-1}(\frac{\sqrt{6}+\sqrt{2}}{4})=\frac{\pi}{4}

Usar o resultado da primeira questão é um complicação desnecessária quando:

\begin{array}{rl}f^{-1}(\dfrac{1}{2}) + g^{-1}(\dfrac{\sqrt{6}+\sqrt{2}}{4})=&f^{-1}(\sin{\dfrac{\pi}{6}})+g^{-1}(\dfrac{\sqrt{6}+\sqrt{2}}{4})\\[\bigskipamount]=&\dfrac{\pi}{6} +g^{-1}\Big(\dfrac{1}{\sqrt{2}}\cdot\dfrac{\sqrt{3}}{2}+\dfrac{1}{\sqrt{2}}\cdot\dfrac{1}{2}\Big )\\[\bigskipamount]=&\dfrac{\pi}{6} +g^{-1}\Big(\cos{\dfrac{\pi}{4}}\cos{\dfrac{\pi}{6}}+\sin{\dfrac{\pi}{4}}\sin{\dfrac{\pi}{6}\Big)\\
[\bigskipamount]=&\dfrac{\pi}{6} +g^{-1}\big (\cos{(\dfrac{\pi}{4}-\dfrac{\pi}{6})}\big)\\
[\bigskipamount]=&\dfrac{\pi}{6} +\dfrac{\pi}{12}\\[\bigskipamount]=&\dfrac{\pi}{4}\end{array}

Ou, usando o resultado de a):

\begin{array}{rl}\cos{\Big (f^{-1}\big(\dfrac{1}{2}\big)+g^{-1}\big(\dfrac{\sqrt{6}+\sqrt{2}}{4}\big)\Big)}=&\sqrt{1-\Big(\dfrac{1}{2}\Big)^2}\cdot \dfrac{\sqrt{6}+\sqrt{2}}{4}-\dfrac{1}{2}\cdot\sqrt{1-\Big(\dfrac{\sqrt{6}+\sqrt{2}}{4}\Big)^2}\\
=&\dfrac{3\sqrt{2}+\sqrt{3}\sqrt{2}}{8}-\dfrac{\sqrt{2}\sqrt{4-2\sqrt{3}}}{8}\\
=&\dfrac{3\sqrt{2}+\sqrt{3}\sqrt{2}}{8}-\dfrac{\sqrt{2}\sqrt{(\sqrt{3}-1)^2}}{8}\\
=&\dfrac{3\sqrt{2}+\sqrt{3}\sqrt{2}-\sqrt{3}\sqrt{2}+\sqrt{2}}{8}=\dfrac{4\sqrt{2}}{8}=\dfrac{1}{\sqrt{2}}\\
=&\cos{\dfrac{\pi}{4}}
\end{array}

\cos{\Big (f^{-1}\big(\dfrac{1}{2}\big)+g^{-1}\big(\dfrac{\sqrt{6}+\sqrt{2}}{4}\big)\Big)}=\cos{\dfrac{\pi}{4}}\Rightarrow g\Bigg(\cos{\Big (f^{-1}\big(\dfrac{1}{2}\big)+g^{-1}\big(\dfrac{\sqrt{6}+\sqrt{2}}{4}\big)\Big)}\Bigg)=g\Bigg(\cos{\dfrac{\pi}{4}}\Bigg)\\
\Rightarrow f^{-1}\big(\dfrac{1}{2}\big)+g^{-1}\big(\dfrac{\sqrt{6}+\sqrt{2}}{4}\big)=\dfrac{\pi}{4}

M03

Sejam C um subconjunto não vazio e P um ponto, ambos em um mesmo plano, tais que P\notin C, Diz-se que "P enxerga C sob um ângulo \alpha se \alpha for a medida do menor ângulo com vértice em P que contenha C.

a) Se C for um circulo de raio r , centrado na origem de um plano cartesiano real, determine o lugar geométrico dos pontos que enxergam C sob um ângulo de 60 graus

\text{Seja O o centro de }\mathcal{C}\text{. As duas retas }P_1,P_2\text{  compondo o \^angulo }\alpha\text{ s\~ao tangentes de }\mathcal{C}.
\text{Sejam A e B os pontos de intersec\c c\~ao entre }P_1,P_2\; e\;C \text{.Sabemos que ent\~ao }(PA)\;e\;(OA)\\\text{  s\~ao perpendiculares.}
\text{O \'e equidistante de }P_1\; e\;P_2\;\;(|OA|=|OB|=r) \text{,e ent\~ao PO \'e bissetriz de }\alpha,\\ \text{e }\widehat{OPA}=\frac{\alpha}{2}.


\text{No triangulo OPA temos ent\~ao }\sin{\frac{\alpha}{2}}=\frac{|OA|}{|OP|},\;ou\;|OP|=\frac{|OA|}{\sin{\frac{\alpha}{2}}}=\frac{r}{\sin{\frac{\alpha}{2}}}.

\text{P pertence ao c\'irculo }\mathcal{C}(O,\frac{r}{\sin{\frac{\alpha}{2}}})\text{ de centro O e de raio }\frac{r}{\sin{\frac{\alpha}{2}}}=\frac{r}{\sin{\frac{\pi}{6}}}=2r

b) Se \mathcal{C} for a união dos segmentos OA e OB em que O=(0,0), A=(a,0) e B=(0,b), com a,b>0, determine o lugar geométrico dos pontos que enxergam \mathcal{C} sob um ângulo de 90 graus.

  • \text{Se }x>0,y>0\text{ ou }x<0,y<0
    \alpha=\widehat{APB}\\
\text{no triangulo APB temos } AP^2+BP^2=AB^2\\
\begin{array}{rl}\text{e }AP^2+BP^2=AB^2 \Leftrightarrow&y^2+(x-a)^2+(y-b)^2+x^2=a^2+b^2\\
\Leftrightarrow&x^2-2\dfrac{a}{2}x+y^2-2\dfrac{b}{2}y=0\\
\Leftrightarrow&(x-\dfrac{a}{2})^2+(y-\dfrac{b}{2})^2=\dfrac{a^2+b^2}{4}\\
\Leftrightarrow&P\in \Bigg (\mathcal{C}\Big((\dfrac{a}{2},\dfrac{b}{2}),\dfrac{\sqrt{a^2+b^2}}{2}\Big)_{x>0,y>0}\bigcup\mathcal{C}\Big((\dfrac{a}{2},\dfrac{b}{2}),\dfrac{\sqrt{a^2+b^2}}{2}\Big)_{x<0,y<0}\Bigg )\\&\text{arcos de c\`irculo de centro }(\dfrac{a}{2},\dfrac{b}{2})\text{ e de raio }\dfrac{\sqrt{a^2+b^2}}{2}\\&\text{ com }(x>0,y>0)\text{ ou }(x<0,y<0)\end{array}
    \text{E j\'a que: }
    \begin{array}{rl} \left \begin{array}{rl}x<0\\y<0 \end{array}\right \} &\Rightarrow \left \{ \begin{array}{rl}(x-\dfrac{a}{2})>\dfrac{a^2}{4}\\[\bigskipamount](y-\dfrac{b}{2})>\dfrac{b^2}{4} \end{array} \right \\[\bigskipamount]&\Rightarrow \big (x-\dfrac{a}{2}\big )^2+\big (y-\dfrac{b}{2}\big )^2>\dfrac{a^2+b^2}{4}\\[\bigskipamount]&\Rightarrow \mathcal{C}\Big((\dfrac{a}{2},\dfrac{b}{2}),\dfrac{\sqrt{a^2+b^2}}{2}\Big)_{x<0,y<0}=\emptyset ,\end{array}\\

    \text{temos } P\in \mathcal{C}\Big((\dfrac{a}{2},\dfrac{b}{2}),\dfrac{\sqrt{a^2+b^2}}{2}\Big)_{x>0,y>0}

  • \text{Se }x\leq 0,y\geq 0
    \alpha=\widehat{BPO}\\
\text{no triangulo OPB temos } BP^2+OP^2=OB^2\\
\begin{array}{rl}\text{e }BP^2+OP^2=OB^2 \Leftrightarrow&x^2+(y-b)^2+x^2+y^2=b^2\\
\Leftrightarrow&x^2+(y-\dfrac{b}{2})^2=\dfrac{b^2}{4}\\
\Leftrightarrow&P\in \mathcal{C}\Big((0,\dfrac{b}{2}),\dfrac{b}{2}\Big)_{x\leq 0,y\geq 0}\end{array}

  • \text{Se }x>0,y<0
    \alpha=\widehat{APO}\\
\text{no triangulo OPA temos } AP^2+OP^2=OA^2\\
\begin{array}{rl}\text{e }AP^2+OP^2=OA^2 \Leftrightarrow&(x-a)^2+y^2+x^2+y^2=a^2\\
\Leftrightarrow&(x-\dfrac{a}{2})^2+y^2=\dfrac{a^2}{4}\\
\Leftrightarrow&P\in \mathcal{C}\Big((\dfrac{a}{2},0),\dfrac{a}{2}\Big)_{x>0,y<0}\end{array}

\text{Seja E o conjunto dos pontos P que enxergam OA e OB sob o \^angulo }\alpha,\\\
E=\mathcal{C}\Big((\dfrac{a}{2},\dfrac{b}{2}),\dfrac{\sqrt{a^2+b^2}}{2}\Big)_{x>0,y>0}\bigcup\mathcal{C}\Big((0,\dfrac{b}{2}),\dfrac{b}{2}\Big)_{x\leq 0,y\geq 0}\end{array}\bigcup\mathcal{C}\Big((\dfrac{a}{2},0),\dfrac{a}{2}\Big)_{x>0,y<0}\end{array}


M04

Considere a sequência a_1=6, a_2=4, a_3=1, a_4=1 e a_n=a_{n-4} para n\geq 5. Defina S_n^k=a_n+...+a_{n+k} para k\geq 0, isto é a soma de (k+1) termos consecutivos da sequência começanco no n-ésimo, por exemplo S_2^1-4+1=5

a) Encontre n,k tais que S_n^k=20

S_6^6=20

b) Para cada inteiro j, 1\leq j \leq 12, encontre n,k tais que S_n^k=j

\begin{array}{ccl>{\Tiny}}j&n&k&S_n^k\\\hline
1&3&0&a_3=1\\
2&4&0&a_4=2\\
3&3&1&a_3=1\\
4&2&0&a_2=4\\
5&2&1&a_2+a_1=4+1=5\\
6&1&0&a_1=6\\
7&2&2& a_2+a_3+a_4=4+1+2=7\\
8&4&1&a_4+a_5=2+6=8\\
9&3&2&a_3+a_4+a_5=1+2+6=9\\
10&1&1&a_1+a_2=6+4=10\\
11&1&2&a_1+a_2+a_3=6+4+1=11\\
12&4&2&a_4+a+5+a_6=2+6+4=12\end{array}


3) Mostre que para qualquer inteiro j, j\geq 1, existem inteiros n\geq 1,k\geq 0 tais queS_n^k=j

\forall j \in \mathBB{N}^*, \exists p,q \in \mathbb{N} \slash j=p\times 13 +q,\text{com }0\leq q<13\\

\text{Ao mesmo tempo, }\forall t\in \mathbb{N}^*, S_t^{3}=13\text{ e } 1\leq q\leq 12 \Rightarrow \exists (u,v) \in \mathbb{N}^*\!\times\! \mathbb{N}\;\slash\; q=S_u^v

\text{Temos ent\~ao }:\\\begin{array}{rl}j=p\times 13+S_u^v=&\displaystyle\sum_{i=0}^{p-1} S_{u+v+1+4i}^{3}+\sum_{i=u}^{u+v} a_i\\
=&\displaystyle \sum_{i=u+v+1}^{u+v+1+4(p-1)+3}\!\!\!\!\!\!\!\!\! \!\!\!\!a_i\;\;\;\;\;\;+\sum_{i=u}^{u+v} a_i\\
=&\displaystyle \sum_{i=u}^{u+v+4p} \!\!\!\!\!\!a_i=S_{u}^{v+4p}\\
=&S_n^k \text{ com }n=u\text{ e }k=v+4p\end{array}\\
\\
\text{Demostramos que }\forall j\in\mathbb{N}^*,\;\exists (n,k)\in \mathbb{N}^*\!\!\!\times\!\!\mathbb{N}\;\slash\;j=S_n^k

M05

Para responder aos itens a) e b) considere a figura correspondente

a) num tetraedro OABC, os ângulos \widehat{AOB}, \widehat{BOC} e \widehat{COA} medem 90 graus. Sejam \alpha e \beta as medidas dos ângulos \widehat{ACO} e \widehat{BCO}, respetivamente,expresse o cosseno do ângulo \widehat{ACB} em função de \alpha e \beta

Imagem




\text{Seja }\gamma=\widehat{ACB}. \text{ No triangulo ACB temos }cos{\gamma}=\dfrac{CE}{BC}=\dfrac{CD}{AC};\\
\text{no triangulo OAC temos }\cos{\alpha}=\dfrac{CE}{OC}=\dfrac{OC}{AC};\\
\text{no triangulo OBC temos }\cos{\beta}=\dfrac{CD}{OC}=\dfrac{OC}{BC}\\
\\
\text{e ent\~ao } \cos{\gamma}=\dfrac{CE}{BC}=\dfrac{OC\cos{\alpha}}{BC}=\dfrac{OC\cos{\alpha}}{\dfrac{OC}{\cos{\beta}}}=\cos{\alpha}\cdot\cos{\beta}

b) Um navio parte do ponto de latitude 0° e de longitude 0°e navega até chagar ao ponto de latitude 45° sul e longitude 45° oeste, seguindo a trajetória que minimiza a distância percorrida. Admita que a terra seja esférica de raio R=6000km. Qual foi a distância percorrida pelo navio?

Imagem

\text{Estamos no caso em que: }
  • C\text{ \'e o centro do globo terrestre}
  • CA=CB=6000\;km
  • \alpha=\beta=\dfrac{\pi}{4}

\cos{\gamma}=\cos{\alpha}\cdot\cos{\beta}=\big(\dfrac{1}{\sqrt{2}}\big)^2=\dfrac{1}{2}\\
\text{e ent\~ao }\gamma=\dfrac{\pi}{3}

\text{O arco de c\'irculo de centro C e de raio igual \`a 6000 km percorrido tem comprimento de:}\\
(6000\times 2\times \pi)\times \dfrac{\dfrac{\pi}{3}}{2\pi}=\dfrac{6000\times 2\times \pi}{6}=2000\times \pi \approx 6280 km


M06

Considere a função real definida por f(x)=\sqrt{x-\dfrac{1}{x}}+\sqrt{1-\dfrac{1}{x}}-x

a) Qual é o domínio de f?

f(x)\;existe\;\Leftrightarrow &\left \{ \begin{array}{l} x\neq 0\\x-\dfrac{1}{x}\geq 0\\1-\dfrac{1}{x}\geq 0 \end{array} \right\\

x-\dfrac{1}{x}\geq 0 \Leftrightarrow \dfrac{x^2-1}{x} \geq 0 \Leftrightarrow \left \{\begin{array}{l}x^2-1\geq 0\;se\;x> 0\\x^2-1\leq 0\;se\;x< 0 \end{array} \right\Leftrightarrow \left \{\begin{array}{l}x\geq 1\;se\;x> 0\\x\geq -1\;se\;x< 0 \end{array} \right\Leftrightarrow x\in [-1;0[\cup]1;+\infty[

1-\dfrac{1}{x}\geq 0 \Leftrightarrow 1 \geq \dfrac{1}{x} \Leftrightarrow \left \{\begin{array}{l}x\geq 1\;se\;x> 0\\x\leq 1\;se\;x< 0 \end{array} \right\Leftrightarrow x\in ]-\infty;0[\cup[1;+\infty[

\mathcal{D}_f = \mathbb{R}^*\;\; \cap\;\; \big ([-1;0[\cup]1;+\infty[\big )\;\; \cap \;\;\big (]-\infty;0[\cup[1;+\infty[\big )=[-1;0[\cup[1;+\infty[



b) Encontre o(s) valor(es) de x para o(s) qual(is) f(x)=0


x\in [-1;0[ \Rightarrow \left \{\begin{array}{l}-x>0\\\sqrt{x-\dfrac{1}{x}}+\sqrt{1-\dfrac{1}{x}}>0 \end{array} \right \Rightarrow f(x)>0\\
\\
\text{f n\~ao tem raiz em }[-1;0[

\text{Seja }x\in [1;+\infty[\\
\\
\begin{array}{rl} f(x)=0 \Leftrightarrow &\sqrt{x-\dfrac{1}{x}}=x-\sqrt{1-\dfrac{1}{x}}\\
\Leftrightarrow & x-\dfrac{1}{x}=x^2+1-\dfrac{1}{x}-2x\sqrt{1-\dfrac{1}{x}}\\
\Leftrightarrow & x^2-x+1=2x\sqrt{1-\dfrac{1}{x}}\\
\Leftrightarrow &x^4-2x^3+3x^2-2x+1=4x^2-4x\\
\Leftrightarrow &x^4-2x^3-x^2+2x+1=0\\
\Leftrightarrow & x^2+\dfrac{1}{x^2} -2(x-\dfrac{1}{x})-1=0\\
\Leftrightarrow & \big (x-\dfrac{1}{x}\big )^2 +2-2(x-\dfrac{1}{x})-1=0\\
\Leftrightarrow &  X^2 -2X+1=0 \text{ com }X=x-\dfrac{1}{x}\\
\Leftrightarrow &  \big (X-1\big )^2=0 \Leftrightarrow X=1\\
\Leftrightarrow & x-\dfrac{1}{x}=1\\
\Leftrightarrow & x^2-x-1=0\\
\Leftrightarrow & x=\dfrac{1+\sqrt{5}}{2} \text{    , j\'a que }\dfrac{1+\sqrt{5}}{2}\in [1;+\infty[\text{ e }\dfrac{1-\sqrt{5}}{2}\notin [1;+\infty[\\
\end{array}


\text{f admite }\dfrac{1+\sqrt{5}}{2}\text{ como \'unica raiz}
rcompany
Novo Usuário
Novo Usuário
 
Mensagens: 6
Registrado em: Seg Fev 18, 2019 00:57
Formação Escolar: PÓS-GRADUAÇÃO
Área/Curso: administração
Andamento: formado

Voltar para Vestibulares

 



  • Tópicos relacionados
    Respostas
    Exibições
    Última mensagem

Quem está online

Usuários navegando neste fórum: Nenhum usuário registrado e 1 visitante

 



Assunto: Princípio da Indução Finita
Autor: Fontelles - Dom Jan 17, 2010 14:42

Não sei onde este tópico se encaixaria. Então me desculpem.
Eu não entendi essa passagem, alguém pode me explicar?
2n \geq n+1 ,\forall n \in\aleph*
O livro explica da seguinte forma.
1°) P(1) é verdadeira, pois 2.1 \geq 1+1
2°) Admitamos que P(k), k \in \aleph*, seja verdadeira:
2k \geq k+1 (hipótese da indução)
e provemos que 2(k+1) \geq (K+1)+1
Temos: (Nessa parte)
2(k+1) = 2k+2 \geq (k+1)+2 > (k+1)+1


Assunto: Princípio da Indução Finita
Autor: MarceloFantini - Seg Jan 18, 2010 01:55

Boa noite Fontelles.

Não sei se você está familiarizado com o Princípio da Indução Finita, portanto vou tentar explicar aqui.

Ele dá uma equação, no caso:

2n \geq n+1, \forall n \in \aleph^{*}

E pergunta: ela vale para todo n? Como proceder: no primeiro passo, vemos se existe pelo menos um caso na qual ela é verdadeira:

2*1 \geq 1+1

Portanto, existe pelo menos um caso para o qual ela é verdadeira. Agora, supomos que k seja verdadeiro, e pretendemos provar que também é verdadeiro para k+1.

\mbox{Suponhamos que P(k), }k \in \aleph^{*},\mbox{ seja verdadeiro:}
2k \geq k+1

\mbox{Vamos provar que:}
2(k+1) \geq (k+1)+1

Daí pra frente, ele usou o primeiro membro para chegar em uma conclusão que validava a tese. Lembre-se: nunca saia da tese.

Espero ter ajudado.

Um abraço.


Assunto: Princípio da Indução Finita
Autor: Fontelles - Seg Jan 18, 2010 02:28

Mas, Fantini, ainda fiquei em dúvida na passagem que o autor fez (deixei uma msg entre o parêntese).
Obrigado pela ajuda, mesmo assim.
Abraço!


Assunto: Princípio da Indução Finita
Autor: Fontelles - Qui Jan 21, 2010 11:32

Galera, ajuda aí!
Por falar nisso, alguém conhece algum bom material sobre o assunto. O livro do Iezzi, Matemática Elementar vol. 1 não está tão bom.


Assunto: Princípio da Indução Finita
Autor: MarceloFantini - Qui Jan 21, 2010 12:25

Boa tarde Fontelles!

Ainda não estou certo de qual é a sua dúvida, mas tentarei novamente.

O que temos que provar é isso: 2(k+1) \geq (k+1)+1, certo? O autor começou do primeiro membro:

2(k+1)= 2k+2

Isso é verdadeiro, certo? Ele apenas aplicou a distributiva. Depois, partiu para uma desigualdade:

2k+2 \geq (k+1)+2

Que é outra verdade. Agora, com certeza:

(k+1)+2 > (k+1)+1

Agora, como 2(k+1) é \geq a (k+1)+2, e este por sua vez é sempre > que (k+1)+1, logo:

2(k+1) \geq (k+1)+1 \quad \mbox{(c.q.d)}

Inclusive, nunca é igual, sempre maior.

Espero (dessa vez) ter ajudado.

Um abraço.


Assunto: Princípio da Indução Finita
Autor: Caeros - Dom Out 31, 2010 10:39

Por curiosidade estava estudando indução finita e ao analisar a questão realmente utilizar a desigualdade apresentada foi uma grande sacada para este problema, só queria tirar uma dúvida sobre a sigla (c.q.d), o que significa mesmo?


Assunto: Princípio da Indução Finita
Autor: andrefahl - Dom Out 31, 2010 11:37

c.q.d. = como queriamos demonstrar =)


Assunto: Princípio da Indução Finita
Autor: Abelardo - Qui Mai 05, 2011 17:33

Fontelles, um bom livro para quem ainda está ''pegando'' o assunto é:'' Manual de Indução Matemática - Luís Lopes''. É baratinho e encontras na net com facilidade. Procura também no site da OBM, vais encontrar com facilidade material sobre PIF... em alguns sites que preparam alunos para colégios militares em geral também tem excelentes materiais.


Assunto: Princípio da Indução Finita
Autor: MarceloFantini - Qui Mai 05, 2011 20:05

Abelardo, faz 1 ano que o Fontelles não visita o site, da próxima vez verifique as datas.


Assunto: Princípio da Indução Finita
Autor: Vennom - Qui Abr 26, 2012 23:04

MarceloFantini escreveu:Abelardo, faz 1 ano que o Fontelles não visita o site, da próxima vez verifique as datas.

Rpz, faz um ano que o fulano não visita o site, mas ler esse comentário dele enquanto respondia a outro tópico me ajudou. hAUEhUAEhUAEH obrigado, Marcelo. Sua explicação de indução finita me sanou uma dúvida sobre outra coisa. :-D